Derivación explícita de la amplitud de Feynman de e+e−→μ+μ−e+e−→μ+μ−e^+e^-\rightarrow\mu^+\mu^-

Estoy tratando de calcular la amplitud de Feynman del proceso

mi + ( pag 1 , s 1 ) mi ( pag 2 , s 2 ) m + ( q 1 , r 1 ) m ( q 2 , r 2 ) ,

considerando como interacción lagrangiana

L I = λ mi ϕ ( X ) ψ ¯ ( X ) ψ ( X ) λ m ϕ ( X ) x ¯ ( X ) x ( X ) ,
dónde ψ es el campo de una partícula escalar H , ψ de mi y x de m .

Usando el teorema de Wick obtengo que la contribución a la amplitud de transición es

S = 2 λ mi λ m 2 d 4 X 1 d 4 X 2 [ ϕ ( X 1 ) ϕ ( X 2 ) ] x ¯ ( X 1 ) x ( X 1 ) ψ ¯ ( X 2 ) ψ ( X 2 ) ,

donde el plazo contratado [ ϕ ( X 1 ) ϕ ( X 2 ) ] = i Δ F ( X 1 X 2 ) .

Entonces

S = 2 λ mi λ m 2 d 4 X 1 d 4 X 2 F | x ¯ ( X 1 ) x ( X 1 ) | 0 0 | ψ ¯ + ( X 2 ) ψ + ( X 2 ) | I i Δ F ( X 1 X 2 ) .

Escribiendo S = ( 2 π ) 4 d 4 ( pag 1 + pag 2 q 1 q 2 ) i η , Lo entiendo

i η = λ mi λ m tu ¯ r 2 ( q 2 ) v r 1 ( q 1 ) v ¯ s 1 ( pag 1 ) tu s 2 ( pag 2 ) i ( pag 1 + pag 2 ) 2 metro 2 + i ε .

Ahora me gustaría hacer el cuadrado de la amplitud en las 16 posibles configuraciones de helicidad en el centro de masa ( pag 1 = pag 2 y q 1 = q 2 ), pero si trato de calcular el producto de los tu y v espinores (incluso antes de hacer el cuadrado), obtengo que todos son cero.

Tengo que hacer (y lo hice) los cálculos usando los espinores:

tu ± ( pag ) = ( mi | pag | ξ pag ± mi ± | pag | ξ pag ± )

v ± ( pag ) = ± ( mi ± | pag | ξ pag mi | pag | ξ pag ) ,

dónde

ξ pag + = ( C o s θ 2 mi i ϕ s i norte θ 2 )
y
ξ pag = ( mi i ϕ s i norte θ 2 C o s θ 2 ) .

Leí en Peskin Schroeder que

tu r ( pag ) v s ( pag ) = 0

entonces puede parecer que todas las amplitudes de esa forma son cero en el centro de masa... ¿Qué me estoy perdiendo?

Esto suena como una pregunta de "revisar mi trabajo", que se considera fuera de tema en este sitio.
¿Suena mejor ahora?

Respuestas (1)

¡Tu error se esconde a simple vista! En tu amplitud tienes aportes de la forma

v ¯ s 1 ( pag ) tu s 2 ( pag )

Pero luego tratas de usar una fórmula para v en vez de v ¯ ! Recuerda eso v ¯ difiere de v por un factor (crucial) de γ 0 . Esto tiene el efecto de hacer

v ¯ s 1 ( pag ) tu s 2 ( pag ) 0

De hecho, la combinación que se desvanece que involucra variables barradas en lugar de dagadas es

v ¯ s 1 ( pag ) tu s 2 ( pag ) = 0

como se prueba en la página 104 de estas excelentes notas .

Sí, lo siento, la cosa con el era solo una suposición. Pero entonces (lo comprobaré de nuevo) no hay ninguna razón por la cual la amplitud deba ser cero, ¿verdad?
Ok, descubrí el problema mientras volvía a pensar en los espinores con dagas/barras gracias a su respuesta: es posible que haya hecho los cálculos sin el γ 0 ... así que usando daggered en lugar de barred
Sí, exactamente: la amplitud no debe ser cero.
Luego obtengo que, por encima de las 16 configuraciones de helicidad, solo 4 no se desvanecen: lo que parece ser correcto considerando que el escalar tiene giro 0. Pero, ¿es correcto que las amplitudes no dependan de los ángulos de dispersión sino solo del módulo de la dos momentos?
¿Estás seguro de que tu | pag | en sus definiciones de tu y v ¿es correcto? En general, esto no es correcto; consulte, por ejemplo, la ecuación (4.115) aquí . Por supuesto, la pregunta podría simplemente querer que mire un caso especial. Si observa el escenario general y calcula la sección transversal no polarizada, esperaría cierta dependencia angular ya que obtendría términos como pag q . Véase, por ejemplo, este ejemplo análogo de QED.
Sí, para el caso que me interesa, estoy seguro de las definiciones. Pensé que la ausencia de dependencia del ángulo podría deberse al hecho de que estamos tratando con una partícula escalar (no una de espín-1).